Please confirm topic selection

Are you sure you want to trigger topic in your Anconeus AI algorithm?

Please confirm action

You are done for today with this topic.

Would you like to start learning session with this topic items scheduled for future?

Review Question - QID 214947

In scope icon M 6 A
QID 214947 (Type "214947" in App Search)
A 33-year-old woman presents to the emergency department with muscle spasms. She has noticed "twitching" in both her upper and lower extremities over the past 3 days that have interfered with her training for a marathon. She has felt more fatigued over the past month. She has no significant medical history. She takes no medication other than whey protein supplements, a multivitamin, and fish oil. Her temperature is 98.5°F (36.9°C), blood pressure is 103/71 mmHg, pulse is 105/min, respirations are 11/min, and oxygen saturation is 100% on room air. Physical exam reveals a thin woman with thinning hair. She appears pale and her mucous membranes are dry. Her neurological and cardiac exams are unremarkable. Laboratory studies are ordered as seen below.

Hemoglobin: 10 g/dL
Hematocrit: 30%
Leukocyte count: 6,100/mm^3 with normal differential
Platelet count: 187,500/mm^3

Serum:
Na+: 130 mEq/L
Cl-: 100 mEq/L
K+: 2.3 mEq/L
HCO3-: 28 mEq/L
BUN: 40 mg/dL
Glucose: 79 mg/dL
Creatinine: 0.9 mg/dL
Ca2+: 8.2 mg/dL
Mg2+: 1.8 mg/dL
Thyroid stimulating hormone (TSH): 4.0 mIU/L
Free T4: 0.4 ng/dL (normal: 0.7-1.53 ng/dL)

The patient is given 4 liters of lactated ringer solution and 40 mEq of potassium. Her repeat whole blood potassium 3 hours later is 2.4 mEq/L, and her repeat BUN is 20 mEq/L. Another 40 mEq of potassium is administered. Another repeat whole blood potassium is 2.5 mEq/L. The patient’s muscle spasms persist. Which of the following is the most likely underlying etiology for this patient’s chief concern?